How to solve this maths equation???​ pls help me, its urgent!!! asap

How To Solve This Maths Equation??? Pls Help Me, Its Urgent!!! Asap

Answers

Answer 1

Answer:

x = [tex]\frac{3wyz}{4wz+wy-2yz}[/tex]

Step-by-step explanation:

Given

[tex]\frac{2}{w}[/tex] + [tex]\frac{3}{x}[/tex] - [tex]\frac{4}{y}[/tex] = [tex]\frac{1}{z}[/tex]

Multiply through by wxyz to clear the fractions

2xyz + 3wyz - 4wxz = wxy ( subtract wxy from both sides )

2xyz + 3wyz - 4wxz - wxy = 0 ( subtract 3wyz from both sides )

2xyz - 4wxz - wxy = - 3wyz ( factor out x from each term on the left side )

x(2yz - 4wz - wy) = - 3wyz ( divide both sides by (2yz - 4wz - wy )

x = [tex]\frac{-3wyz}{2yz-4wz-wy}[/tex] ( multiply numerator/ denominator by - 1 )

x = [tex]\frac{3wyz}{4wz+wy-2yz}[/tex]


Related Questions

Solve the simultaneous equation
X+3y=13
X-y=5

Answers

Answer:

x = 7

y = 2

Step-by-step explanation:

In the above question, we are given 2 equations which are simultaneous. To solve this equation, we have to find the values of x and y

x + 3y = 13 ........ Equation 1

x - y = 5...........Equation 2

From Equation 2,

x = 5 + y

Substitute 5 + y for x in Equation 1

x + 3y = 13 ........ Equation 1

5 + y + 3y = 13

5 + 4y = 13

4y = 13 - 5

4y = 8

y = 8/4

y = 2

Since y = 2, substitute , 2 for y in Equation 2

x - y = 5...........Equation 2

x - 2 = 5

x = 5 + 2

x = 7

Therefore, x = 7 and y = 2

If EH = 23, calculate AB.


Youngblood say you want me back in your life...

Answers

Answer:

2/4 = 23/AB

1/2 = 23/AB

AB= 46

Hope it helps ^_^

Evaluate the expression for the given value of the variable. −3x3, when x = 4

Answers

Answer:

The answer is - 192

Step-by-step explanation:

The expression

- 3x³

To find the value of the expression when

x = 4 substitute the value of x that's 4 into the expression

We have

- 3(4)³

= - 3( 64)

The final answer is

- 192

Hope this helps you

In the first quadrant you start at 5, 6 and move 4 units down. What point will you end up at? Thanks for your help! - Someone who's better at English than math

Answers

Answer:

(5, 2)

Step-by-step explanation:

(5, 6) go down 4 units means subtract 4 from the y

(5, 2)

The point to end up will be (5, 2).

What is Coordinates?

A pair of numbers which describe the exact position of a point on a cartesian plane by using the horizontal and vertical lines is called the coordinates.

Given that;

In the first quadrant you start at (5, 6 ) and move 4 units down.

Now,

Since, In the first quadrant you start at 5, 6 and move 4 units down.

Hence, The end up point = (5, 6 - 4)

                                       = (5, 2)

Thus, The point to end up will be (5, 2).

Learn more about the coordinate visit:

https://brainly.com/question/18269861

#SPJ5

Given that p=x^2-y^2/x^2+xy
I. Express p in the simplest form
ii. Find the value of p if x=-4 and y=-6

Answers

Answer:

p = - [tex]\frac{1}{2}[/tex]

Step-by-step explanation:

Given

p = [tex]\frac{x^2-y^2}{x^2+xy}[/tex] ( factorise numerator and denominator )

x² - y² ← is a difference of squares and factors as (x - y)(x + y)

x² + xy ← factor out x from each term

= x(x + y) , thus

p = [tex]\frac{(x-y)(x+y)}{x(x+y)}[/tex] ← cancel (x + y) on numerator/ denominator

   = [tex]\frac{x-y}{x}[/tex] ← substitute x = - 4, y = - 6

   = [tex]\frac{-4-(-6)}{-4}[/tex]

   = [tex]\frac{-4+6}{-4}[/tex]

    = [tex]\frac{2}{-4}[/tex] = - [tex]\frac{1}{2}[/tex]

A classroom floor has an area of
(30x^3 + 8x^2, with a width of 2x feet.
What is the length of the floor?

Answers

Answer:

15x² + 4x feet

Step-by-step explanation:

We need to calculate (30x³ + 8x²) / 2x.

(30x³ + 8x²) / 2x

= 30x³ / 2x + 8x² / 2x

= 15x² + 4x

7 liters of gasoline between their 4 cars. How many liters of gasoline should each car get?

Answers

Answer:

1.75 liters for each car.

Step-by-step explanation:

Divide:

*7 / 4 = 1.75.

Check:

*1.75 x 4 = 7.

1.75
7 divided by 4

Write an equation for the line in the graph that passes through the points (0,4) and (12,16).

Answers

Answer:

We have,

y-y1=m(x-x1)

or,y-4=-1(x-0)

or,y-4=-x

or,x+y=4 is the required equation

Step-by-step explanation:

it it helps u ...plz mark it as brainliest

Which number line represents the solution set for the inequality –negative StartFraction one-half EndFraction x is greater than or equal to 4.x ≥ 4?

A number line from negative 10 to 10 in increments of 2. A point is at negative 2 and a bold line starts at negative 2 and is pointing to the left.
A number line from negative 10 to 10 in increments of 2. A point is at negative 8 and a bold line starts at negative 8 and is pointing to the left.
A number line from negative 10 to 10 in increments of 2. A point is at negative 2 and a bold line starts at negative 2 and is pointing to the right.
A number line from negative 10 to 10 in increments of 2. A point is at negative 8 and a bold line starts at negative 8 and is pointing to the right.

Answers

Answer:

it's b :)

Step-by-step explanation:

A number line which represents the solution set for the given inequality is: option B.

What is a number line?

A number line refers to a type of graph with a graduated straight line which contains numerical values (both positive and negative numbers) that are placed at equal intervals along its length.

Next, we would solve the given inequality:

-½x ≥ 4

-x ≥ 4 × 2

x ≤ -8.

Therefore, a number line which represents the solution set for the given inequality is a number line from -10 to 10 in increments of 2 with a point at -8 and a bold line starts at -8 while pointing to the left.

Read more on number line here: brainly.com/question/24644930

#SPJ9

The base of a triangle is two times its height. If the area of the triangle is 36, then what is the height of the triangle?

Answers

We have:

h - height

b = 2h - base

A = 36 - area

so:

[tex]A=\frac{1}{2}\cdot b\cdot h\\\\A=\frac{1}{2}\cdot 2\cdot h \cdot h\\\\A=h^2\\\\36=h^2\quad|\sqrt{(\dots)}\\\\\boxed{h=6}[/tex]

find the lower quartile for the data {47.2, 33.8, 43, 62, 5.8, 9, 61.4, 30.8, 68.2, 51.6, 13.2, 17.4, 64.2, 50.6, 29.4, 40.4}

Answers

Answer:

The lower quartile is 23.4

Step-by-step explanation:

The given data are;

47.2, 33.8, 43, 62, 5.8, 9, 61.4, 30.8, 68.2, 51.6, 13.2, 17.4, 64.2, 50.6, 29.4, 40.4

Rearranging the data, we have;

5.8, 9, 13.2, 17.4, 29.4, 30.8, 33.8, 40.4, 43, 47.2, 50.6, 51.6, 61.4, 62, 64.2, 68.2

The lower quartile, Q₁, is the (n + 1)/4 th term which is (16 +1)/4 = 4.25th term

However since we have an even set of numbers, we place a separator at the middle and we look for the median of the left half as follows

5.8, 9, 13.2, 17.4, 29.4, 30.8, 33.8, 40.4║ 43, 47.2, 50.6, 51.6, 61.4, 62, 64.2, 68.2

We have two numbers (17.4 + 29.4) at the median of the left set of numbers, we find the average of the two numbers to get the lower quartile

The lower quartile is therefore = (17.4 + 29.4)/2 = 23.4.

PLEASE HELP ME WORTH 20 POINTS It looks like the graph of the parents function f(x)x^2. However:
- It has been reflected (flipped) over the x-axis
-It has been shifted down 4 units.
-It had been shifted left 1 unit

Step 1: Start with the equation f(x) = x2. Write the equation for the graph of g(x) that has been reflected, or flipped, over the x-axis.

Step 2: Use the equation you wrote in Step 1. Write the equation for the graph of g(x) that has also been shifted down 4 units.

Step 3: Use the equation you wrote in Step 2. Write the equation for the graph of g(x) that has also been shifted left 1 unit.

Answers

flipped : [tex]-x^2[/tex]

moving down: [tex] -x^2+4[/tex]

shifting left [tex] -(x+1)^2+4[/tex]

expanding it: [tex] -x^2-2x+3[/tex]

Answer:

1. f(x)=x^2

f(x)=-x^2

2. f(x)=-x^2-4

3. f(x)=-(x+1)^2-4

Will Give Brainliest, Answer ASAP m∠O =
m∠N =

Answers

Answer:

∠ O = 61°, ∠ N = 119°

Step-by-step explanation:

In a parallelogram

Consecutive angles are supplementary

Opposite angles are congruent, thus

x + 2x - 3 = 180

3x - 3 = 180 ( add 3 to both sides )

3x = 183 ( divide both sides by 3 )

x = 61°

Thus

∠ O = ∠ M = x = 61°

∠ N = ∠ P = 2x - 3 = 2(61) - 3 = 122 - 3 = 119°

two similar cups are 3 cm and 5 cm deep if the larger cup
s hold 675 cm cube of water what is the volume of the smaller one​

Answers

Answer:

145.8

Step-by-step explanation:

l.s.f for the two is 3:5

volume scale factor will be 3³:5³ which us 27:125

so 27×675 / 125

= 145.8

Find the slope of the line that passes through the points (-8,-3) and (2, 3)
0
1
3/5
5/3

Answers

Answer:

The answer is

[tex] \frac{3}{5} [/tex]

Step-by-step explanation:

To find the slope passing through two points we use the formula

[tex]m = \frac{y2 - y1}{x2 - x1} [/tex]

Where

m is the slope

( x1 , y1) and ( x2 , y2) are the points

From the question the points are

(-8,-3) and (2, 3)

So the slope is

[tex]m = \frac{3 + 3}{2 + 8} = \frac{6}{10} = \frac{3}{5} [/tex]

Hope this helps you

On a coordinate plane, a triangle has points (negative 5, 1), (2, 1), (2, negative 1).
Use the drop downs to answer the following questions about the distance between the points (−5, 1) and (2, −1).

What is the distance of the horizontal leg?

What is the distance of the vertical leg?

Use the Pythagorean theorem. What is the distance between the two points?

Answers

Answer:

The answer is below

Step-by-step explanation:

The points of the triangle are  (- 5, 1), (2, 1), (2, - 1). The distance between two points is given by:

[tex]Distance=\sqrt{(x_2-x_1)^2+(y_2-y_1)^2}[/tex]

The horizontal leg is formed by points with the same y axis. Therefore the points that make up the horizontal leg is (- 5, 1), (2, 1). The Distance of the horizontal leg is:

[tex]Horizontal\ leg=\sqrt{(2-(-5))^2+(1-1)^2}=\sqrt{7^2+0}=7\ units[/tex]

The vertical leg is formed by points with the same x axis. Therefore the points that make up the vertical leg is (2 1), (2, 1-). The Distance of the vertical leg is:

[tex]Vertical\ leg=\sqrt{(2-2)^2+(-1-1)^2}=\sqrt{0+(-2)^2}=2\ units[/tex]

The hypotenuse is gotten using Pythagorean theorem. It is gotten by:

Hypotenuse² = (Horizontal leg)² + (Vertical leg)²

Hypotenuse² = 7² + 2²

Hypotenuse² = 49 + 4 = 53

Hypotenuse = √53

Hypotenuse = 7.28 unit

Answer:

The answer are 7, 2 and 53

Step-by-step explanation:

Each packet of the cooking oil weighs 2/5th of a kilogram and one kilogram of the cooking oil costs $6.5. Sara went to the grocery shop to buy some items to stock her kitchen. If she bought 8 packets of the cooking oil, how much money did she spend? A $19.60 B $18.20 C $20.80 D $23.40

Answers

Answer:

C) $20.80

Step-by-step explanation:

1 kg of cooking oil = $6.5

1 packet of cooking oil =2/5 kg

If 1 kg of cooking oil = $6.5

2/5kg of cooking oil = $X

Cross Multiply

1kg × $X = 2/5kg × $6.5

$X = 13/5

$X = 2.6

Hence 2/5kg of oil cost $2.6

Since 1 packet of oil = 2/5kg of oil , 1 packet of oil cost $2.6

The amount she spent if she bought if she bought 8 packets of the cooking oil is calculated as:

1 packet of oil = $2.6

8 packets of oil =

$2.5 × 8

= $20.80

Therefore,if Sara bought 8 packets of oil, the amount she would spend = $20.80

[fill in the blank]
In this figure,AB and CD are parallel.

AB is perpendicular to line segment_____. If the length of EF is a units, then the length of GH is_____units.

Answers

Answer:

1. GH

2. a

Step-by-step explanation:

Perpendicular: When 2 lines meet at 90 degrees

1. It is line segment GH because AB and GH meet at a 90 degree angle (since there is a box at angle GHF indicating that it is 90 degrees)

2. It has to be a units because it is a rectangle where the top and bottom are congruent and the sides are too

This is a rectangle since AB and CD are parallel and GH can be a transversal line, according to same side interior angles theorem EGH is a also 90 degrees. That means FEG is 90 degrees too because then the quadrilateral will add up to 360 degrees

how many 6-digit numbers can be created using8, 0, 1, 3, 7, and 5 if each number is used only once?

Answers

Answer:

600 numbers

Step-by-step explanation:

For six-digit numbers, we need to use all digits 8,0,1,3,7,5 each once.

However, 0 cannot be used as the first digit, because it would make a 5-digit number.

Therefore

there are 5 choices for the first digit (exclude 0)

there are 5 choices for the first digit (include 0)

there are 4 choices for the first digit

there are 3 choices for the first digit

there are 2 choices for the first digit

there are 1 choices for the first digit

for a total of 5*5*4*3*2*1 = 600 numbers

Which equation represents the line that is perpendicular to y=3/4x+1 and passes through (-5,11)
Will give brainliest!!

Answers

Answer:

y = - [tex]\frac{4}{3}[/tex] x + [tex]\frac{13}{3}[/tex]

Step-by-step explanation:

The equation of a line in slope- intercept form is

y = mx + c ( m is the slope and c the y- intercept )

y = [tex]\frac{3}{4}[/tex] x + 1 ← is in slope- intercept form

with slope m = [tex]\frac{3}{4}[/tex]

Given a line with slope m then the slope of a line perpendicular to it is

[tex]m_{perpendicular}[/tex] = - [tex]\frac{1}{m}[/tex] = - [tex]\frac{1}{\frac{3}{4} }[/tex] = - [tex]\frac{4}{3}[/tex] , thus

y = - [tex]\frac{4}{3}[/tex] x + c ← is the partial equation

To find c substitute (- 5, 11) into the partial equation

11 = [tex]\frac{20}{3}[/tex] + c ⇒ c = 11 - [tex]\frac{20}{3}[/tex] = [tex]\frac{13}{3}[/tex]

y = - [tex]\frac{4}{3}[/tex] x + [tex]\frac{13}{3}[/tex] ← equation of perpendicular line

The equation of the line that passes through (-5, 11) and perpendicular to y = (3/4)x + 1 is

y = -2x + 1

What is an equation of a line?

The equation of a line is given by:

y = mx + c

where m is the slope of the line and c is the y-intercept.

Example:

The slope of the line y = 2x + 3 is 2.

The slope of a line that passes through (1, 2) and (2, 3) is 1.

We have,

y = (2/4)x + 1 is in the form of y = m(2)x + c

So,

m(2) = 2/4 = 1/2

The equation of the line y = m(1)x + c is perpendicular to y = (2/4)x + 1.

So,

m(1) x m(2) = -1

m(1) = -1/(1/2)

m(1) = -2

Now,

y = -2x + c passes through (-5, 11).

This means,

11 = -2 x (-5) + c

11 = 10 + c

11- 10 = c

c = 1

Thus,

The equation of the line is y = -2x + 1.

Learn more about equation of a line here:

https://brainly.com/question/23087740

#SPJ2

Given f(x) = 2x - 7, complete parts (a) through (c).
A. Solve f(x)=0.
B. What do the answers to parts (a) and (b) tell you about the graph of y=f(x)

Answers

Answer:

a) x=7/2

Step-by-step explanation:

a) since f(x) is=0, plug in 0 to → f(x)=2x-7 [this f(x)]. you would get 0=2x-7. solve for x by adding 7 and dividing by 2 which you get x=7/2.

Then value of [tex]x[/tex] is 7/2

What is function?

Functions are the fundamental part of the calculus in mathematics. The functions are the special types of relations. A function in math is visualized as a rule, which gives a unique output for every input . Mapping or transformation is used to denote a function in math. These functions are usually denoted by letters. The domain is defined as the set of all the values that the function can input while it can be defined. The range is all the values that come out as the output of the function involved. Co-domain is the set of values that have the potential of coming out as outputs of a function.

given function:

[tex]f(x)[/tex]= 2[tex]x[/tex] -7

So,[tex]f(x)[/tex]= 0

2[tex]x[/tex] -7=0

2[tex]x[/tex]= 7

[tex]x[/tex]= 7/2

The graph is attached below.

Learn more about function here:

https://brainly.com/question/12431044

#SPJ2

Brian invested his savings in two investment funds. The $8000 that he invested in Fund A returned a 4% profit. The amount that he invested in Fund B returned a 1% profit. How much did he invest in Fund B, if both funds together returned a 2% profit?

Answers

Answer: Brian invested $16000 in Fund B .

Step-by-step explanation:

Let x be the amount Brian invested in Fund B.

Given, The $8000 that he invested in Fund A returned a 4% profit. The amount that he invested in Fund B returned a 1% profit.

i.e. profit on Fund A = 4% of 8000 = 0.04 ×8000 = $320

Profit on Fund B = 1% of x = 0.01x

Together they earn 1% profit, i.e. Combined profit = 2% of (8000+x)

= 0.02(8000+x)

As per question,

Combined profit=Profit on Fund A+Profit on Fund B

[tex]\Rightarrow\ 0.02(8000+x) =320+0.01x\\\\\Rightarrow\ 0.02(8000) +0.02x=320+0.01x\\\\\Rightarrow\ 160+0.02x=320+0.01x\\\\\Rightarrow\ 0.02x-0.01x=320-160\\\\\Rightarrow\ 0.01x=160\\\\\Rightarrow\ x=\dfrac{160}{0.01}\\\\\Rightarrow\ x=16000[/tex]

Hence, Brian invested $16000 in Fund B .

convert the equation f(x)=1/2x^2+3x-2 to vertex form

Answers

Answer:

Step-by-step explanation:

Hello, please consider the following.

The "vertex form" is as below.

[tex]y=a(x-h)^2+k\\\\\text{Where (h, k) is the vertex of the parabola.}\\[/tex]

Let's do it!

[tex]f(x)=\dfrac{1}{2}x^2+3x-2\\\\f(x)=\dfrac{1}{2}\left(x^2+3*2*x\right) -2\\\\f(x)=\dfrac{1}{2}\left( (x+3)^2-3^2\right)-2\\\\f(x)=\dfrac{1}{2}(x+3)^2-\dfrac{9}{2}-\dfrac{4}{2}\\\\f(x)=\dfrac{1}{2}(x+3)^2-\dfrac{9+4}{2}\\\\\large \boxed{\sf \bf \ \ f(x)=\dfrac{1}{2}(x+3)^2-\dfrac{13}{2} \ \ }[/tex]

Thank you.

Anyone want to help...?

Answers

Answer:

-1

Step-by-step explanation:

3/2 * (-22/33)

Simplify by dividing the second fraction by 11

3/2 * (-2/3)

Rewriting

3/3 * (-2/2)

-1/1

Answer:

-1

Step-by-step explanation:

(a/b)(c/d) = (a*c)(

(3/2)(-22/33)

(3*-22)/(2*33) = -66/66 = -1

PLS HELP. i really need this fast ill give brainliest too

Answers

Answer:

24 square units

Step-by-step explanation:

Use the formula for area of a parallelogram to solve.  The base is 6 units, and the height is 4 units.

A = bh

A = (6)(4)

A = 24 square units

The area of the parallelogram is 24 square units.

If a polygon has an area of 10 cm² and is dilated by a factor of 2, what will be the area of the dilated polygon?

Answers

Area depends on the product of sides,

so if the sides are shortened by a factor of 2, area will reduce by a factor of 4. (2×2)

new area = 10/4=2.5 cm²

Suppose y varies jointly as x & z. If y = -180 when z = 15and x = -3,then find y when x = 7 and z = -5. 1 point

Answers

Answer:

y = - 140

Step-by-step explanation:

The statement

y varies jointly as x & z is written as

y = kxz

to find y when x = 7 and z = -5 we must first find the relationship between the variables

when y = - 180

z = 15

x = - 3

- 180 = k(15)(-3)

-180 = - 45k

Divide both sides by - 45

k = 4

The formula for the variation is

y = 4xz

when

x = 7

z = -5

y = 4(7)(-5)

y = 4(-35)

y = - 140

Hope this helps you

(b) The train is 61 cm long and travels at a speed of 18 cm/s.
It takes 4 seconds for the whole of the train to cross a bridge.
Calculate the length of the bridge.​

Answers

Answer:

The length of the bridge is 72 cm

Step-by-step explanation:

In order to find the length of bridge, we have to apply distance formula which is D = S × T where S represents speed and T is time :

[tex]d = s \times t[/tex]

[tex]let \: s = 18,t = 4[/tex]

[tex]d = 18 \times 4[/tex]

[tex]d = 72 \: cm[/tex]

The length of the bridge is 11 cm .

What is relationship between distance time and speed ?

When an object moves in a straight line at a steady speed, we can calculate its speed if we know how far it travels and how long it takes. This equation shows the relationship between speed, distance traveled and time taken:

Speed is distance divided by the time taken.

For example, a car travels 30 kilometers in 2 hours.

Its speed is 30 ÷ 2 = 15km/hr.

Formula used :

Distance  = Speed * Time

Time = Distance / Speed

Speed = Distance / Time

According to the question

Length of train = 61 cm

Speed of train = 18 cm/s

Time taken to cross the bridge = 4 seconds

In this length traveled by train = length of train + Length of bridge

( as time given is to completely cross platform )

Therefore,

length traveled by train = 61 + Length of bridge

formula used

Distance  = Speed * Time

61 + Length of bridge =  18 * 4

61 + Length of bridge = 72

Length of bridge  = 72 - 61

Length of bridge  = 11 cm

Hence, the length of the bridge is 11 cm .

To know more about relationship between distance time and speed  here :

https://brainly.com/question/26754391

# SPJ2

HELP i don’t know how to do this

Answers

Answer:

4a

Step-by-step explanation:

4a

the top and right are a-b, but you have to add the b’s back in, so really all sides are a

a+a+a+a=4a

Answer: 4a

Step-by-step explanation: perimeter is the length of all sides added together. Every length is given combine like variables and you will get 4a+2b-2b. 2b-2b is 0 which leaves you with 4a

how many eighth rests are in a half rest?

Answers

If I’m not mistaking there’s 4, In other words two eighth rests make up a quarter rest, while four of them make up a half rest, and eight 1/8 notes make up a whole rest.
Other Questions
Which sentence uses an apostrophe correctly?Penelopes' children are staying over tonight.Phillip's car broke down on the highway.Papas tacos' are the best I've ever tasted.Pams plates' have been washed and put away. : A spaceship is traveling at the speed 2t 2 1 km/s (t is time in seconds). It is pointing directly away from earth and at time t 0 it is 1000 kilometers from earth. How far from earth is it at one minute from time t 0 Find the sum of the first 10 terms of the following geometric sequences: 3,6,12,24,48 Arizona Senate Bill 1070, often referred to colloquially as the "papers, please" law, was aimed widely at those of: what influences the costumer when she or he is deciding whether to buy a product Use the gradient to find the directional derivative of the function at P in the direction of Q. g(x, y, z) = xye^z, P(2, 4, 0), Q(0, 0, 0) Monochromatic light is incident on a pair of slits that are separated by 0.220 mm. The screen is 2.60 m away from the slits. (Assume the small-angle approximation is valid here.) (a) If the distance between the central bright fringe and either of the adjacent bright fringes is 1.97 cm, find the wavelength of the incident light. (b) At what angle does the next set of bright fringes appear? Pls answer I really need helpBrainlist and thank you will be the reward thank you so much!!! Construct a polynomial function with the following properties: fifth degree, 2 is a zero of multiplicity 2, 5 is the only other zero, leading coefficient is 3. A local museum charges $25 per adult and $12 per child for admission fees. At the end of a day, the museum made $9,014 in total admission revenue, not including sales tax, and had a total of 450 guests. The system of equations below can be used to model the number of guests that were children, x, and the number of guests that were adults, y. Howcanwe minimizethe effect of disaster? Fill in the blank in the following sentence with the appropriate preterite verbbelow.Yoa la playa con mi familia el verano pasado.O A. voy B. fuiO C. fueO D. soy Find the value of x in the triangle Discuss how the idea of vertical farming reflects the role of science in society. You decide to invest in a portfolio consisting of 30 percent Stock A, 30 percent Stock B, and the remainder in Stock C. Based on the following information, what is the expected return of your portfolio? State of Economy Probability of State Return if State Occurs of Economy Stock A Stock B Stock C Recession .17 - 18.8 % - 3.9 % - 22.8 % Normal .45 10.2 % 8.5 % 17.1 % Boom .38 28.6 % 15.8 % 31.7 % Choose all that apply. In what areas did major changes take place during the Industrial Revolution? textiles aviation mining oil production manufacturing In which mountain range can you find Rich Mountain the highest peak in Oklahoma Region? A. the Ouachita Mountains B. the Ozark Mountains C. the Wichita Mountains D. the Arbuckle Mountains what is the coefficient of x in the equation of 32+2x=10 solve after finding the coefficient Work out the value of angle x Suppose that prices of a certain model of new homes are normally distributed with a mean of $150,000. Use the 68-95-99.7 rule to find the percentage of buyers who paid: between $150,000 and $152,400 if the standard deviation is $1200. A. 68% B. 99.7% C. 47.5% D. 34%